-2

Is $\Bbb Q^+/\langle2,3\rangle$ not Hausdorff?

Let $\Bbb Q^+$ have the standard topology inherited from $\Bbb R$

Let $/\langle2,3\rangle$ indicate the quotient taken by setting $x\sim y\iff x\cdot\{2^m3^n:m,n\in\Bbb Z\}=y\cdot\{2^m3^n:m,n\in\Bbb Z\}$

Looking at the definition of Hausdorff (for the first time!), I think it's NOT because $\log_2(3)$ is irrational so $\langle2,3\rangle$ is dense in $\Bbb Q^+$ and therefore for every neighbourhood of any $x$ I can always find an element $h$ of $\langle2,3\rangle$ such that $hy$ is arbitrarily close to $x$. Is that right?

Fine, I know a fix but I would really appreciate a bit of a hand to write it properly.

Question

How do I modify $\Bbb Q^+/\langle2,3\rangle$ to make it Hausdorff?

Attempt

I know the solution I need is to introduce an order relation based on the 3-adic valuation:

$x\prec y\iff \lvert x\rvert_3>\lvert y\rvert_3$

And I know that $x\in\Bbb Q^+/\langle2\rangle$ approaches a limit $y$ if $\lvert \nu_3(x)-\nu_3(y)\rvert=1$ and $\lvert x-y\rvert\to 0$

And I can give an example as $n\to\infty$:

$x+(1-2^{-6n})\cdot2^{\nu_2(x)}\cdot3^{\nu_3(x)-1}\to x+2^{\nu_2(x)}\cdot3^{\nu_3(x)-1}$

e.g. $1\langle2\rangle\to\frac{85}{64}\langle2\rangle\ldots\frac43\langle2\rangle$

Then finally take the quotient using $\langle3\rangle$ to finish up.

How do I write / define this topology properly and clearly? I'm not sure whether to talk of the order relation $\succ$ and an order topology or to build if $\lvert \nu_3(x)-\nu_3(y)\rvert=1$ into some metric.

Robert Frost
  • 9,620
  • 1
    You are right, and basically give the right reason, that the standard topology one would put on that quotient, i.e. the quotient topology coming from the standard topology on $\mathbb Q^+$, is not Hausdorff. – Torsten Schoeneberg Jul 29 '20 at 22:58
  • 2
    Of course one can endow that set with many topologies which are Hausdorff (e.g. the discrete topology). The subtle point would be to make precise what other properties you want that topology to have. Should it be metrizable? What algebraic/arithmetic structure of that quotient is it supposed to respect? I see your attempt but cannot make much sense of it. In particular, anything that comes from a $3$-adic valuation seems rather pointless if one mods out powers of $3$. – Torsten Schoeneberg Jul 29 '20 at 23:05
  • @TorstenSchoeneberg the point will be that it gives us Zorn's lemma on the Collatz graph. – Robert Frost Jul 30 '20 at 01:09
  • Haha, now it's clear what to look for ... – Torsten Schoeneberg Jul 30 '20 at 03:29
  • @TorstenSchoeneberg I was addressing your claim it seemed pointless. I'll sort out clarity of the convergent sequence in due course. I worked out why it's so obvious it converges. It's because it converges in the overriding space so one can define the order and the convergent sequences there and the quotient can be applied afterwards. – Robert Frost Jul 30 '20 at 07:33
  • @TorstenSchoeneberg the only properties i need it to have are that sequences of the form I give converge, and that the elements of $\Bbb Q^+/\langle2,3\rangle$ are Hausdorff. No further algebraic properties. This is only about separability of the $5$-rough representatives and convergence of sequences of the form I give. – Robert Frost Jul 30 '20 at 10:43

1 Answers1

2

Too long to put in the comment box:

You got the set (multiplicative monoid actually) $X:= \mathbb Z [\frac16]^+$, and your function $f:X \rightarrow X$ on it, good.

It's also good you can compute the $n$-fold composition $f^n$ explicitly. It is true and neat that for all $x \in X$,

$$\lim_{n\to \infty} f^n(x) = x +2^{v_2(x)}3^{v_3(x)-1}$$

where the limit is taken w.r.t. the topology coming from $X \subset \mathbb R$.

On the other hand, it is true that $f(2x) =f(3x)=f(x)$ for all $x \in X$. This implies that $f$ naturally induces a function on the quotient $Q := X /\langle 2, 3 \rangle$, i.e.

$$\tilde f (x\cdot \langle 2,3 \rangle) := f(x) \cdot \langle 2, 3 \rangle$$

is well-defined and gives us a function $\tilde f :Q \rightarrow Q$, very good.

But to speak of convergence of any sequence in $Q$, one has to put a topology on $Q$.

The only natural candidate for that is the quotient topology. Alas, that topology is the indiscrete (sometimes called "trivial") topology, where the only open sets are the empty set and the full set $Q$.

In this topology, every sequence converges to any point ("at the same time").

So even though it is indeed true that

$$\lim_{n\to\infty}\tilde f^n(x\cdot \langle 2,3\rangle)= (x +2^{v_2(x)}3^{v_3(x)-1} )\cdot \langle 2, 3 \rangle,$$

this is true for the trivial reason that

$$\lim_{n\to\infty}\tilde f^n(x\cdot \langle 2,3\rangle)= a\cdot \langle 2, 3 \rangle$$

for all $a \in X$.

To see the issue in a clearer but similar example: Let $X = \mathbb R^+$. Take the quotient $Q:= \mathbb R^+/\mathbb Q^+$. Look at the sequence $$\sqrt2, \quad 1.4 \sqrt2, \quad 1.41\sqrt2, \quad 1.414\sqrt2, \quad 1.4142\sqrt2 \quad ...$$ You will agree that in $X$, this sequence converges to $2$. Now look at the corresponding sequence in the quotient $Q$. You will want to say, and it might be reasonable to assume, that it still converges to $2$ or rather, to its residue class $2\cdot \mathbb Q$ (which happens to be just $\mathbb Q$). But also, the sequence in the quotient is just $$\sqrt2 \cdot \mathbb Q, \quad \sqrt2 \cdot \mathbb Q, \quad \sqrt2 \cdot \mathbb Q, \quad \sqrt2 \cdot \mathbb Q, \quad\sqrt2 \cdot \mathbb Q \quad ...$$ because all those factors $1, 1.4, 1.41,...$ were rational so get "swallowed" by $\mathbb Q$. So the sequence is actually constant, so it is also very reasonable to say it converges to the non-trivial residue $\sqrt 2 \cdot \mathbb Q$. So there you have it already converging to two different points in the quotient, and so far we have not even chosen a topology on $Q$, just demanded that it satisfies the very reasonable assumptions

a) If $\lim_{n\to \infty} a_n = a$ in $X$, then $\lim_{n\to \infty} \bar a_n = \bar a$ in $Q$.

b) For a constant sequence $(\bar a, \bar a, \bar a ,...)$ in $Q$, we have $\lim_{n\to \infty} \bar a = \bar a$.

Assumption b) is satisfied by any topology. (And by the way, assumption a) is equivalent to demanding that

a') The projection map $X \twoheadrightarrow Q$ is continuous.)

So here's your dilemma in a nutshell: Either you want to be able to conclude from convergence in $X$ to convergence of residues in $Q$, via assumption a). Then you have to live with the fact that in both your and my example, by virtue of the modded out subset being dense, this actually forces each sequence in $Q$ to converge to all points in Q simultaneously, i.e. the notion of convergence becomes totally meaningless.

But if you choose a topology on $Q$ in which it is a non-trivial statement to say that some sequence converges to some point (in particular if you want limits to be unique if they exist, i.e. the topology to be Hausdorff): then there will be sequences in $X$ which converge to a certain limit $L$ there, but whose projections in $Q$ will not converge to the projection of the limit $\bar L$.


I repeat, there are many (I guess uncontably many) topologies on $Q$ which are Hausdorff. But the above shows that for none of them, it is automatic that you can just the say the projection of a limit is the limit of the projections. I do not know if it is possible at all, and if it is, one needs to to put in significant work, to ensure that for all your sequences of interest $f^n(x)$ that property a) would still hold. This could be kind of an interesting question:

Is there a Hausdorff topology on $Q$ such that $\lim \tilde f^n(x\langle 2,3\rangle) = (x +2^{v_2(x)}3^{v_3(x)-1} )\cdot \langle 2, 3 \rangle$ for all $x \in X$, or at least for an interesting subset, like your $5$-rough integers.

  • Thank-you for this. Your suggestion at the end is indeed what I'm trying to get at. I half "have" the answer already which my "attempt" at the end of my question tries to communicate. The powers of $2$ in $f^n(x)$ decrease as $n$ increases and the sequence converges to a number with a 3-adic valuation one lower than the terms of the sequence. So we first need the topology of $\Bbb R$ on the numbers $\lvert x\rvert_3=1$, then we can glue on inductively (with a $\succ$) the "lower bounds" which satisfy $\lvert x\rvert_3=3$. It works out $x+2^{\nu_2(x)}3^{\nu_3(x)-1}$ surjects over those... – Robert Frost Jul 30 '20 at 19:21
  • ...so we get all of ${x:\lvert x\rvert_3=3}$. So with ${x:\lvert x\rvert_3=n}\succ{x:\lvert x\rvert_3=3n}$ we cover all the numbers. But I'm not familiar with the order topology or whether this idea hangs together formally. – Robert Frost Jul 30 '20 at 19:21
  • I'm starting to think this: https://math.stackexchange.com/q/3776055/334732 is a prerequisite for such a topology. – Robert Frost Jul 31 '20 at 19:43
  • If $v(x)=x\cdot \lvert x\rvert_3$ then let $v_2(x)=\dfrac{v(x)}{\min{2^n:2^n>x}}$ then $d(x,y)=\lvert v_2(x)-v_2(y)\rvert$ gives us the Hausdorff topology we require, right? – Robert Frost Aug 01 '20 at 20:35
  • No. Neither $v$ nor that minimum are well-defined on $Q$. – Torsten Schoeneberg Aug 02 '20 at 01:12
  • maybe I wrote it badly but $x\lvert x\rvert_2$ finds the odd factor times $\langle3\rangle$. I guess $v(x)={v=x/\langle2\rangle:\lvert v\rvert_2=1}$ is better expressed. The sequences converge with this metric in the sense that successors get closer together, but I think the limit points still need glueing on somehow. – Robert Frost Aug 02 '20 at 04:36
  • 1
    What you "guess is better expressed" makes absolutely no sense at all, as in: I cannot even read that. If that $d$ is supposed to be some kind of metric on $Q$, my previous comment has not been addressed. If OTOH that $d$ is supposed to be some kind of metric on $X$, I point out $d(1,2)=0$. Even if that's fixed, I don't believe the triangle inequality until I see a proof. Even if that is done, I would need to be convinced that this metric does induce something non-trivial on the quotient. – Torsten Schoeneberg Aug 02 '20 at 05:13
  • it's not on $X$ it's on $Q$ so $\forall x\in X\forall y\in \langle2,3\rangle: d(x,xy)=0$ is what we want. You just gave an example of what we DO want. Also triangle inequality checks out. And sequences accumulate. But I'm unclear on to what. FWIW the metric can be thought of as projecting down to the 5 rough representatives and measuring only those. – Robert Frost Aug 02 '20 at 05:21
  • the problem here is the comment format. $v:Q\to X/\langle3\rangle$ and as for $v_2$ apply it element-wise and you see it sends every element to the same place, i.e. the odd integer divided by the power of 2 that puts it in the interval $[1/2,1)$ so we have $\langle2\rangle\to1/2$ – Robert Frost Aug 02 '20 at 06:05
  • I'm sorry, it should have said :If $v(x)=x\cdot \lvert x\rvert_3$ then let $v_2(x)=\dfrac{v(x)}{\min{2^n:2^n>\color{red}{v}(x)}}$ then let $d(x,y)=\lvert v_2(x)-v_2(y)\rvert$ – Robert Frost Aug 02 '20 at 22:52
  • FWIW I have re-worked it to make convergence clearer by dodging the quotient issue somewhat. It's the same problem essentially but hopefully this fixed the convergence problem: https://math.stackexchange.com/questions/3778091/ – Robert Frost Aug 02 '20 at 23:17